Eufisky - The lost book

2018年考研试题

证明积分不等式:
 
\[\frac{1}{5}<\int_0^1\frac{xe^xdx}{\sqrt{x^2-x+25}}<\frac{2}{\sqrt{99}}.\]
 
证明  注意到
\[x^2-x+25=\left(x-\frac{1}{2}\right)^2+\frac{99}{4}>\frac{99}{4}, a.e. x\in [0,1],\]
从而
\[\int_0^1\frac{xe^xdx}{\sqrt{x^2-x+25}}<\frac{2}{\sqrt{99}}\int_0^1xe^xdx=\frac{2}{\sqrt{99}}.\]
另一方面, 分部积分, 得到
\[\int_0^1\frac{xe^xdx}{\sqrt{x^2-x+25}}=\frac{(x-1)e^x}{\sqrt{x^2-x+25}}\Big|_0^1+\int_0^1\frac{(x-1)(x-\frac{1}{2})e^x}{\sqrt{(x^2-x+25)^3}}dx=\frac{1}{5}+\int_0^1\frac{(x-1)(x-\frac{1}{2})e^x}{\sqrt{(x^2-x+25)^3}}dx.\]
\[f(x)=\frac{x-\frac{1}{2}}{\sqrt{(x^2-x+25)^3}},g(x)=(x-1)e^x,\]
\[f'(x)=\frac{-2x^2+2x+\frac{97}{4}}{\sqrt{(x^2-x+25)^5}}>0,\forall x\in [0,1],\int_0^1 f(x)dx=0, g'(x)=xe^x,\]
因此$f,g$在$[0,1]$上严格递增. 根据Chebyshev 积分不等式,
\[\int_0^1\frac{(x-1)(x-\frac{1}{2})e^x}{\sqrt{(x^2-x+25)^3}}dx=\int_0^1f(x)g(x)dx>\int_0^1 f(x)dx\int_0^1g(x)dx=0.\]
\[\int_0^1\frac{xe^xdx}{\sqrt{x^2-x+25}}>\frac{1}{5}.\]
最后得到
\[\frac{1}{5}<\int_0^1\frac{xe^xdx}{\sqrt{x^2-x+25}}<\frac{2}{\sqrt{99}}.\]

2018年武汉大学653数学分析
 
一(30分).1.计算极限$$\lim_{n\rightarrow\infty}\sum_{k=n^2}^{(n+1)^2}\frac1{\sqrt k}.$$
    2.计算极限$$\lim_{n\rightarrow\infty}\frac{\int_0^\mathrm\pi\sin^nx\cos^6x\operatorname dx}{\int_0^\mathrm\pi\sin^nx\operatorname dx}$$
    3.已知$x_{n+1}=\ln\left(1+x_n\right)$,且$x_1>0$,计算$$\lim_{n\rightarrow\infty}nx_n$$
 
二.设$f(x),f_1(x)$在$[a,b]$区间上连续,$f_{n+1}(x)=f(x)+\int_a^x\sin\{f_n(t)\}\operatorname dt$,证明:$\{f_n\}$在$[a,b]$一致收敛.
 
 
三.设$$f(x)=\left\{\begin{array}{lc}e^{-\frac1{x^2}}&,\;x\neq0\\0&,\;x=0\end{array}\right.$$证明$f(x)$在$x=0$处任意阶导数存在.
 
 
四.已知$(x_1,x_2,x_3)\in{R}^3$,其中$u=\frac1{\left|x\right|},\left|x\right|=\sqrt{x_1^2+x_2^2+x_3^2}$,计算
$$\oint\limits_S\frac{\partial^2 u}{\partial x_i\partial x_j}{\rm d}S,i,j=1,2,3$$,其中$S:x_1^2+x_2^2+x_3^2=R^2$
 
五.讨论求解方程$f(x)$牛顿切线法.1.推导牛顿切线法迭代公式;
                                                   2.在适当的条件下,证明牛顿切线法收敛
 
六(20分).求极限$$\lim_{n\rightarrow\infty}(nA-\sum_{k=1}^nf(\frac kn))=B$$存在时,$A,B$的值。
 
七.设$u_i=u_i(x_1,x_2),i=1,2$,且关于每个变量为周期1的连续可微函数,求$$\iint\limits_{0\leq x_1,x_2\leq1}det(\delta_{ij}+\frac{\partial u_i}{\partial x_j})dx_1dx_2,$$其中$det(\delta_{ij}+\frac{\partial u_i}{\partial x_j})$是映射$x\mapsto(x_1+u_1,x_2+u_2)$的雅克比行列式.
 
八(40分).设$f(x)$在$[a,b]$上$Riemann$可积,$\varphi(x)$是周期为$T$的连续函数,证明:
        1.存在阶梯函数$g_\varepsilon(x)$使得$$\int_a^b\left|f(x)-g_\varepsilon(x)\right|\operatorname dx<\frac\varepsilon2$$
        2.计算$$\lim_{n\rightarrow\infty}\int_a^b\varphi(nx)\operatorname dx$$
        3.证明$$\lim_{n\rightarrow\infty}\int_a^bf(x)\varphi(nx)\operatorname dx=\frac1T\int_0^T\varphi(x)\operatorname dx\int_a^bf(x)\operatorname dx$$
        4.计算$$\lim_{n\rightarrow\infty}\frac1{\ln n}\int_0^T\frac{\varphi(nx)}xdx,其中函数\frac{\varphi(nx)}x收敛$$

试题(1b):证明\[\lim_{n\to +\infty}\left(\int_0^1\frac{\sin x^n}{x^n}dx\right)^n=\prod_{k=1}^{+\infty}\exp{\left(\frac{(-1)^k}{2k(2k+1)!}\right)}\]
 
作代换$x^n=y$
\[\int_0^1\frac{\sin x^n}{x^n}dx=\frac{1}{n}\int_0^1\frac{\sin y}{y^{2-\frac{1}{n}}}dy\]
\[\int_0^1 y^{\frac{1}{n}-2}\sum_{k=0}^{+\infty}\frac{(-1)^k}{(2k+1)!}y^{2k+1}dy=\int_0^1 y^{\frac{1}{n}-1}dy+\int_0^1\sum_{k=1}^{+\infty}\frac{(-1)^k}{(2k+1)!}y^{2k-1+\frac{1}{n}}dy\]
容易得到
\[\int_0^1 y^{\frac{1}{n}-1}dy=n,\left|\frac{(-1)^k}{(2k+1)!}y^{2k-1+\frac{1}{n}}\right|\le \frac{1}{(2k+1)!}\]
由优级数判别法后一项中的函数项级数一致收敛,可以逐项积分,上式
\[=n+\sum_{k=1}^{+\infty}\frac{(-1)^k}{(2k+1)!}\int_0^1 y^{2k-1+\frac{1}{n}}dy=n+\sum_{k=1}^{+\infty}\frac{(-1)^k}{(2k+1)!}\frac{1}{2k+\frac{1}{n}}\]
最终我们得到
\[\int_0^1\frac{\sin x^n}{x^n}dx=1+\frac{1}{n}\sum_{k=1}^{+\infty}\frac{(-1)^k}{(2k+1)!}\frac{1}{2k+\frac{1}{n}}\]
取对数,要证明的表达式化为
\[\lim_{n\to+\infty}n\ln \left(1+\frac{1}{n}\sum_{k=1}^{+\infty}\frac{(-1)^k}{(2k+1)!}\frac{1}{2k+\frac{1}{n}}\right)=\sum_{k=1}^{+\infty}\left(\frac{(-1)^k}{2k(2k+1)!}\right)\]
\[\left|\sum_{k=1}^{+\infty}\frac{(-1)^k}{(2k+1)!}\frac{1}{2k+\frac{1}{n}}\right|\le \sum_{k=1}^{+\infty}\frac{1}{(2k+1)!}<+\infty\]
\[\lim_{n\to+\infty}\frac{1}{n}\sum_{k=1}^{+\infty}\frac{(-1)^k}{(2k+1)!}\frac{1}{2k+\frac{1}{n}}=0\]
\[\lim_{n\to+\infty}n\ln \left(1+\frac{1}{n}\sum_{k=1}^{+\infty}\frac{(-1)^k}{(2k+1)!}\frac{1}{2k+\frac{1}{n}}\right)=\lim_{n\to+\infty}\sum_{k=1}^{+\infty}\frac{(-1)^k}{(2k+1)!}\frac{1}{2k+\frac{1}{n}}\]
现在我们希望极限与级数和交换顺序,考虑$[0,1]$上的函数项级数
\[f(x)=\sum_{k=1}^{+\infty}\frac{(-1)^k}{(2k+1)!}\frac{1}{2k+x}\]
仍然由优级数判别法一致收敛,故由$\mathrm{Heine}$定理原式
\[=\lim_{n\to +\infty}f(\frac{1}{n})=f(0)=\sum_{k=1}^{+\infty}\frac{(-1)^k}{(2k+1)!}\frac{1}{2k}\]
作者: TangSong    时间: 昨天 02:00
 
试题(1c):证明
 
\[\lim_{n\to +\infty}\frac{1}{n}\sum_{k=1}^n\ln(1+\frac{k^2-k}{n^2})=\ln 2-2+\frac{\pi}{2}\]
注意到下面式子中第一项是一个$\mathrm{Riemann}$和我们有
\[\lim_{n\to +\infty}\frac{1}{n}\sum_{k=1}^n\ln(1+\frac{k^2}{n^2})=\int_0^1\ln(1+x^2)dx=x\ln(1+x^2)\left|_0^1\right.-\int_0^1\frac{2x^2}{1+x^2}dx=\ln 2-2+\frac{\pi}{2}\]
我们只需要再证明
\[\lim_{n\to +\infty}\frac{1}{n}\sum_{k=1}^n\ln(1+\frac{k^2-k}{n^2})=\lim_{n\to +\infty}\frac{1}{n}\sum_{k=1}^n\ln(1+\frac{k^2}{n^2})\]
利用不等式
\[x,y\ge 0,\left|\ln(1+x)-\ln(1+y)\right|=\left|\frac{x-y}{1+\xi}\right|\le|x-y|\]
\[\left|\frac{1}{n}\sum_{k=1}^n\ln(1+\frac{k^2-k}{n^2})-\frac{1}{n}\sum_{k=1}^n\ln(1+\frac{k^2}{n^2})\right|\le \frac{1}{n}\sum_{k=1}^n\left|\ln(1+\frac{k^2-k}{n^2})-\ln(1+\frac{k^2}{n^2})\right|\]
\[\le \frac{1}{n}\sum_{k=1}^n\frac{k}{n^2}=\frac{n+1}{2n^2}\to 0\]
作者: TangSong    时间: 昨天 02:54
试题(8)
$f(x)$在$[1,+\infty)$上二次可导,$\forall x\in [1,+\infty),f(x)>0,f''(x)\le 0,f(+\infty)=+\infty$
证明
\[\lim_{s\to 0^+}\sum_{n=1}^{+\infty}\frac{(-1)^n}{f^s(n)}\]
存在并求之.
 
由二阶导数非正,$f'(x)$在$[1,+\infty)$单减,容易看出$f'$恒正.事实上若有某个$x_0,f'(x_0)\le 0$则由单调性
\[\forall x\ge x_0,f'(x)\le f'(x_0)\le 0,f(x)\le f(x_0)\]与$f(+\infty)=+\infty$矛盾.因此$f$在$[1,+\infty)$严增.
我们将收敛性的证明与求值放在一起进行.
\[S_{2n}(s)=\sum_{k=1}^n \left(\frac{1}{f^s(2k)}-\frac{1}{f^s(2k-1)}\right)\]
注意和式中每个括号都是负的且级数通项趋于$0$,只需要证明对固定的$s>0,S_{2n}(s)$有下界则
\[\lim_{n\to +\infty}S_{2n}(s)\]存在且等于
\[\sum_{n=1}^{+\infty}\frac{(-1)^n}{f^s(n)}\]
由$\mathrm{Lagrange}$中值定理,
\[\frac{1}{f^s(2k)}-\frac{1}{f^s(2k-1)}=\frac{-sf'(\xi)}{f^{s+1}(\xi)},\xi\in (2k-1,2k)\]
注意$f$单增而$f'$单减我们有
\[\frac{-sf'(2k-1)}{f^{s+1}(2k-1)}\le\frac{1}{f^s(2k)}-\frac{1}{f^s(2k-1)}\le \frac{-sf'(2k)}{f^{s+1}(2k)}\]
\[\sum_{k=1}^n\frac{-sf'(2k-1)}{f^{s+1}(2k-1)}\le S_{2n}(s)\le \sum_{k=1}^n\frac{-sf'(2k)}{f^{s+1}(2k)}\]
利用面积原理的思想来估计左右两端.
由单调性$k\ge 2$时
\[\frac{f'(2k-1)}{f^{s+1}(2k-1)}\le \frac{1}{2}\int_{2k-3}^{2k-1} \frac{f'(t)}{f^{s+1}(t)}dt\]
\[\sum_{k=2}^{+\infty}\frac{f'(2k-1)}{f^{s+1}(2k-1)}\le\frac{1}{2}\int_{1}^{+\infty} \frac{f'(t)}{f^{s+1}(t)}dt=\frac{1}{2}\frac{-1}{sf^s(t)}\left|_{t=1}^{t=+\infty}\right.=\frac{1}{2sf^s(1)}\]
 
$S_{2n}(s)$有下界故极限存在.再次利用面积原理
$k\ge 1$时
\[\frac{f'(2k)}{f^{s+1}(2k)}\ge \frac{1}{2}\int_{2k}^{2k+2} \frac{f'(t)}{f^{s+1}(t)}dt\]
\[\sum_{k=1}^{+\infty}\frac{f'(2k)}{f^{s+1}(2k)}\ge\frac{1}{2}\int_{2}^{+\infty} \frac{f'(t)}{f^{s+1}(t)}dt=\frac{1}{2}\frac{-1}{sf^s(t)}\left|_{t=2}^{t=+\infty}\right.=\frac{1}{2sf^s(2)}\]
 
\[-s\left(\frac{f'(1)}{f^{s+1}(1)}+\frac{1}{2sf^s(1)}\right)\le \lim_{n\to +\infty}S_{2n}(s)\le -s\frac{1}{2sf^s(2)}\]
由前面说明就有
\[-s\left(\frac{f'(1)}{f^{s+1}(1)}+\frac{1}{2sf^s(1)}\right)\le \sum_{n=1}^{+\infty}\frac{(-1)^n}{f^s(n)}\le -s\frac{1}{2sf^s(2)}\]
而上式左右两端在$s\to 0^+$时极限都是$-\frac{1}{2}$故
\[\lim_{s\to 0^+}\sum_{n=1}^{+\infty}\frac{(-1)^n}{f^s(n)}=-\frac{1}{2}\] 

中科院2018研究生入学考试 数学分析+高等代数
数学分析部分
 
01. (15pt) 计算极限
\[\lim_{x\to\infty}\left(\sin\frac1x+\cos\frac1x\right)^{x}\text{.}\]
02. (15pt) 计算极限
\[\lim_{x\to 0} \left(\frac{4+\mathrm{e}^{\frac1x}}{2+\mathrm{e}^{\frac4x}}+\frac{\sin x}{|x|} \right)\text{.}\]
03. (15pt) 判断 (并证明) 函数 $f(x,y)=\sqrt{|{xy}|}$ 在点 $(0,0)$ 处的可微性.
 
04. (15pt) 求三个实常数 $a,b,c$,使得下式成立
\[\lim_{x\to 0}\frac1{\tan x -ax}\int_b^x\frac{s^2}{\sqrt{1-s^2}}\,\mathrm{d}s =c\text{.}\]
05. (15pt) 计算不定积分
\[\int\frac{\mathrm{d}x}{\sin^6 x+\cos^6 x}\text{.}\]
06. (15pt) 设函数 $f(x)$ 在 $[-1,1]$ 上二次连续可微,$f(0)=0$,证明:
\[
\left|\int_{-1}^1 f(x)\,\mathrm{d}x\right|\leq\frac{M}{3},\quad \text{其中 }M=\max_{x\in[-1,1]}\left|f''(x)\right|\text{.}
\]
07. (15pt) 求曲线 $y=\dfrac12x^2$ 上的点,使得曲线在该点处的法线被曲线所截得的线段长度最短.
 
08. (15pt) 设 $x>0$,证明
\[\sqrt{x+1}-\sqrt{x}=\frac1{2\sqrt{x+\theta}}\text{,}\]其中 $\theta=\theta(x)>0$,并且 $\lim\limits_{x\to 0}\theta(x)=\dfrac 14$.
 
09. (15pt) 设
\[u_n(x)=\frac{(-1)^n}{(n^2-n+1)^x}\quad (n\geq 0)\text{,}\]求函数 $f(x)=\sum\limits_{n=0}^{\infty}u_n(x)$ 的绝对收敛、条件收敛以及发散的区域.
 
10. (15pt) 证明
\[\frac15<\int_0^1\frac{x\mathrm{e}^x}{\sqrt{x^2-x+25}}\,\mathrm{d}x<\frac{2\sqrt{11}}{33}\text{.}\]
 
高等代数部分
 
一、(20pt) 设 $p(x),q(x),r(x)$ 都是数域 $\mathbb{k}$ 上的正次数多项式,而且 $p(x)$ 与 $q(x)$ 互素,$\mathrm{deg}(r(x))<\mathrm{deg}(p(x))+\mathrm{deg}(q(x))$.证明:存在数域 $\mathbb{k}$ 上的多项式 $u(x),v(x)$,满足 $\mathrm{deg}(u(x))<\mathrm{deg}(p(x)),\,\mathrm{deg}(v(x))<\mathrm{deg}(q(x))$,使得
\[\frac{r(x)}{p(x)q(x)}=\frac{u(x)}{p(x)}+\frac{v(x)}{q(x)}\text{.}\]
二、(20pt) 设 $n$ 阶方阵 $M_n=\left(|i-j|\right)_{1\leq i,j \leq n}$,令 $D_n=\mathrm{det}(M_n)$ ($M_n$ 的行列式).
  (1) 计算 $D_4$;
  (2) 证明 $D_n$ 满足递推关系式 $D_n=-4D_{n-1}-4D_{n-2}$;
  (3) 求 $n$ 阶方阵 $A_n=\left(\left|\frac1i-\frac1j\right|^{\llap{\phantom{b}}}\right)_{1\leq i,j \leq n}$ 的行列式 $\mathrm{det}(A_n)$.
 
三、(20pt) 设 $A,B$ 均是 $n$ 阶方阵,满足 $AB=0$.证明
  (1) $\mathrm{rank}(A)+\mathrm{rank}(B) \leq n$;
  (2) 对于方阵 $A$ 和正整数 $k\,(\mathrm{rank}(A) \leq k \leq n)$,必存在方阵 $B$,使得
\[\mathrm{rank}(A)+\mathrm{rank}(B)=k\text{.}\]
四、(20pt) 通过正交变换将下面的实二次型化成标准型:
\[q(x_1,x_2,x_3)=5x_1^2+5x_2^2+5x_3^2-2x_1x_2-2x_2x_3-2x_1x_3\text{.}\]
五、(20pt) 设 $A$ 和 $B$ 是两个 $n$ 阶实矩阵,并且 $A$ 是对称正定矩阵,$B$ 是反对称矩阵.证明:$A+B$ 是可逆矩阵.
 
六、(20pt) 设 $A$ 是 $n$ 阶复数矩阵,且 $A=\left(\begin{array}{l} A_1\\ A_2\end{array}\right)$,令
\[V_1=\left\{\,x\in\mathbb{C}^n\,\middle|\,A_1 x=0\,\right\},\quad V_1=\left\{\,x\in\mathbb{C}^n\,\middle|\,A_2 x=0\,\right\}\text{,}
\]证明:矩阵 $A$ 可逆的充分必要条件是向量空间 $\mathbb{C}^n$ 能够表示成子空间 $V_1$ 与 $V_2$ 的直和:$\mathbb{C}^n=V_1 \oplus V_2$.
 
七、(15pt) 证明:$8$ 个满足 $A^3=0$ 的 $5$ 阶复数矩阵中必有两个相似.
 
八、(15pt) $\mathbb{R}$ 上所有 $n\,(n\geq 2)$ 阶方阵构成的线性空间 $V=\mathbb{R}^{n \times n}$ 上的线性变换 $f:\, V \to V$ 定义为
\[f(A)=A+A'\quad \forall A\in V\text{,}\]其中 $A'$ 为 $A$ 的转置.求 $f$ 的特征值、特征子空间、极小多项式.

第九题的解答
 
 
9.  设 $B_R=\{(x,y): x^2+y^2< R^2\},u\in C^2( B_R)\cap C(\overline {B_R})$ .
 
1) 若$\Delta u\geqslant 0$, 证明
\[\max_{(x,y)\in\overline {B_R}} u(x,y)= \max_{(x,y) \in \partial B_R} u(x,y).\]
 
证明 对任意$\varepsilon>0$, 令$v_\varepsilon
(x,y)=u(x,y)+\varepsilon (x^2+y^2)$, 则
\[\Delta v_\varepsilon (x,y)=\Delta u(x,y)+4\varepsilon\geqslant 4\varepsilon.\]
由此用反证法易证
\[\max_{(x,y)\in\overline {B_R}} v_\varepsilon (x,y)= \max_{(x,y) \in \partial B_R} v_\varepsilon(x,y).\]
令$\varepsilon\to 0^+$, 即得
\[\max_{(x,y)\in\overline {B_R}} u(x,y)= \max_{(x,y) \in \partial B_R} u(x,y).\]
 
 
2).  若$\Delta u(x,y)=0$, 则
\[\frac{d}{dr}\left(\frac{1}{2\pi r}\int_{\partial B_r}u(x,y)ds\right)=0, 0\leqslant r\leqslant R.\]
 
 
证  注意到
\[\frac{1}{2\pi r}\int_{\partial B_r}u(x,y)ds=\frac{1}{2\pi}\int_0^{2\pi}u(r\cos\theta,r\sin\theta)d\theta=
\int_{\partial B_1}u(rx,ry)ds.\]从而根据Gauss公式, 得到
 
\begin{align*}\frac{d}{dr}\left(\frac{1}{2\pi r}\int_{\partial
B_r}u(x,y)ds\right)&=\frac{1}{2\pi}\int_{\partial B_1}(u_x(rx,ry)x+u_y(rx,ry)y)ds\\
&=\frac{1}{2\pi}\int_{\partial B_1} \frac{\partial
u(rx,ry)}{\partial
\nu}ds\\
&=\frac{1}{2\pi}\iint\limits_{\overline B_1}\Delta
u(rx,ry)dxdy\\
&=0.\end{align*}
3).  证明 若$\Delta u(x,y)=0$, 则
\[u(0,0)=\frac{1}{2\pi r}\int_{\partial B_r}u(x,y)ds.\]
 
证 根据2), 得到
\[\frac{1}{2\pi r}\int_{\partial B_r}u(x,y)ds=\lim_{r\to 0^+}\frac{1}{2\pi r}\int_{\partial B_r}u(x,y)ds=u(0,0).\]

2017-2018学年北京大学高等代数实验班期末试题2018.1.9
2018.1.9 上午8:30--10:30\\

安金鹏

据悉今年使用的教材是 K. Hoffman, R. Kunze: Linear Algebra

反响好我再发出期中试卷

一、设矩阵$A\in \mathbb{R}^{4\times 4}$的矩阵元均为$1$或$-1$, 求$\det A$的最大值.

二、设$V$是所有从有限域$F_p$到自身的映射构成的$F_p$-线性空间. 定义$T, U\in L(V)$为
$$ T(f)(t)=f(-t), \ \  U(f)(t)=f(t+1)-f(t), \ \ \forall \ f\in V, t\in F_p.$$
求$\det T$和$\det U$.

三、设$V$是有限维$F$-空间, $W$是$V$的子空间, $T\in L(V)$满足$T(W)\subset W$. 定义$T_W\in L(W)$和$T_{V/W}\in L(V/W)$为
$$T_W(\alpha)=T(\alpha), \alpha\in W,$$
$$T_{V/W}(\alpha+W)=T(\alpha)+W, \alpha\in V.$$
证明$\det T=\det T_W \det T_{V/W}$.

四、设$A\in F^{n\times n}$, $V$和$W$是$F^n$的子空间. 证明下述等价:

(a) 对任意的$a\in V-\{0\}$, 存在$\beta\in W$使得$\alpha A\beta^t\neq 0$.

(b) 对任意的$\gamma\in F^n$, 存在$\beta\in W$使得对任意的$\alpha\in V$有$\alpha V\beta^t=\alpha\gamma^t$.

五、设$F$是无限域. 证明对多项式代数$F[x]$的任意有限维子空间$V$, 存在$F[x]$的理想$M$满足
$$V\cap M=\{0\}, \ \ \ V+M=F[x].$$

 

关于中科院数学系统院考研事宜(续)

作为跨考过来的一员,也在中科院数学系统院这边呆了一段时间,借着硕转博考试前的一些闲暇时间,再写一篇考研鸡汤。

首先是你得选好报考单位,中国科学院大学(简称国科大)这边有三个数学的机构,包括中国科学院数学与系统科学研究院(也就是我所说的中科院数学系统院,在中关村校区),中国科学院大学数学科学学院(指本部,在玉泉路校区),中科院武汉物理与数学研究所(显然在武汉),关于这几个机构有啥研究方向和导师名单,请有心人自行百度官网查询。另外,这几个机构的数学学生研一都在雁栖湖校区(基础数学研一都在玉泉路校区)进行集中教学,研二才回各自的研究所。

下面我具体介绍下数学系统院这边的情况,这边有四个研究所,分别为数学研究所(数学所),应用数学研究所(应用所),计算数学研究所(计算所),系统科学研究所(系统所)。我就在这个系统所,此所很迷,包括了我们实验室的系统理论和控制论方向,也有统计学,计算机科学,管理科学,金融学等方向。除此之外,吴文俊老先生的数学机械化中心也在我们所,没想到吧,关于这个所,我了解的是他们得学很多代数课程,所以一般也归到基础数学的行列。

如果你很希望加入我们院,成为Xionger的师弟师妹。请大家把握住下面几次机会。第一次,便是数学夏令营,一般在7月底放暑假前的几天吧,这个数学系统院官网会贴通知进行报名。夏令营期间有竞赛考试选拔出前几名,考试内容为数分高代,我微信公众号有这几年的真题。考试没通过的同学会让你进行招生考试或者考研。

第二次,数学系统院会派一些老师去部分985高校进行招生考试,比如说东南大学,厦门大学,中山大学,吉林大学,北京科技大学等等。这个招生考试一般来说是数院院长,副院长那边通知的吧,不会挂到网上,所以如果你是别的学校的,请尽早打探好情报,并问清楚你是否也能参加考试。笔试完还有面试,中科院的面试相对而言还是比较轻松,这边的老师普遍和蔼可亲,以人为本。希望同学们尽可能实事求是,充分发挥出自己的真实水平。

最后一次机会便是考研,虽然说考数分高代,但是高代题稍微难点,数分还可以。由于报考人数多,考研相对而言难度大点,这对跨专业考研的同学来说是福音。另外我也推荐跨考的同学,尽可能报考系统所的有关方向,这是经验也是真相。系统所相对而言比较欢迎跨考的学生,在这里也不是水数学,很多方向研究起来都得用大量的数学,甚至是一些前沿而热门的工具,希望大家不要有所偏见。比如我们控制论方向的,可以参考郭雷老师写的书《控制理论导论》。另外,这边的管理科学,统计学,应用统计,金融学貌似也用大量数学知识,希望经管专业的同学读研前有所准备。

如果你铁了心读基础数学,做黎曼猜测,那中科院是一个很好的选择。请你务必好好学习基础课程,打好数学功底,尽量拿到保研名额,参加各类数学竞赛提高软实力。重要的事情说三遍,尽量保研,保研,保研!

最后说一句,我之所以这么古道热肠,也是因为经历过一些难得的经历,希望大家好好珍惜。情报再多,也比不了你自己去实践,也比不上你的真才实学。但行好事,莫问前程;你若盛开,清风自来!祝好!

 

记国科大一些给力的老师们.

张三国老师在高等数理统计课上说:好,已经走了48个人,我们的目标是再走48个人。同学们戏称为:被三国作业支配的恐惧,三国杀,不是英雄,不读三国。

 

 

2017考研题解

假设$\displaystyle x_0=1,x_n=x_{n-1}+\cos x_{n-1}(n=1,2,\cdots )$,证明:当$x\rightarrow \infty $时, $\displaystyle x_n-\frac{\pi }{2}=o\left(\frac{1}{n^n}\right)$.


证.先证$1\leq x_n<\frac\pi /2$,得到$x_n-x_{n-1}>0$,由单调有界定理可知$x_n$极限存在且$\lim_{n\to\infty}=\frac\pi2$.下面用归纳法证明$\lim_{n\rightarrow\infty}n^n\left(x_n-\frac{\pi}{2}\right)=0$.假设

$$\lim_{n\rightarrow\infty}n^n\left(x_n-\frac{\pi}{2}\right)=0.$$
我们有
\begin{align*}\lim_{n\rightarrow\infty}\left(n+1\right)^{n+1}\left(x_{n+1}-\frac{\pi}{2}\right)&=\lim_{n\rightarrow\infty}\left(n+1\right)^{n+1}\left(x_n+\cos x_n-\frac{\pi}{2}\right)\\&=\lim_{n\rightarrow\infty}\left(n+1\right)^{n+1}\left(x_n+\sin\left(\frac{\pi}{2}-x_n\right)-\frac{\pi}{2}\right)\\&=\lim_{n\rightarrow\infty}\left(n+1\right)^{n+1}\left(x_n+\left(\frac{\pi}{2}-x_n\right)-\frac{1}{6}\left(\frac{\pi}{2}-x_n\right)^3-\frac{\pi}{2}\right)\\&=-\frac{1}{6}\lim_{n\rightarrow\infty}\left(n+1\right)^{n+1}\left(\frac{\pi}{2}-x_n\right)^3\\&=-\frac{1}{6}\lim_{n\rightarrow\infty}\frac{\left(n+1\right)^{n+1}}{n^{3n}}\left[n^n\left(x_n-\frac{\pi}{2}\right)\right]^3=0.\end{align*}

TangSong:令$y_n=\frac\pi2-x_n$,得到$y_n=y_{n-1}-\sin y_{n-1}$.可以证明$$\lim_{n\rightarrow\infty}\frac{y_{n+1}}{y_{n}^{3}}=\frac{1}{6}.$$因此当$n>N$时,我们有$$\frac{y_{n+1}}{y_{n}^{3}}<\frac{1}{2}.$$因此$$0<y_n<\frac{1}{2}y_{n-1}^{3}<\left(\frac{1}{2}\right)^{1+3}y_{n-2}^{3^2}<\cdots <\left(\frac{1}{2}\right)^{1+3+\cdots +3^{n-N-2}}y_{N+1}^{3^{n-N-1}},$$

即$$0<y_n<\left(\frac{1}{2}\right)^{\left(3^{n-N-1}-1\right)/2}y_{N+1}^{3^{n-N-1}}.$$


设$$\left({\begin{array}{*{20}{c}}{{x_{3n}}}\\{{x_{3n + 1}}}\\{{x_{3n + 2}}}\end{array}}\right)=\left({\begin{array}{*{20}{c}}3&{ - 2}&1\\4&{ - 1}&0\\4&{ - 3}&2\end{array}} \right)\left( {\begin{array}{*{20}{c}}{{x_{3n - 3}}}\\{{x_{3n - 2}}}\\{{x_{3n - 1}}}\end{array}} \right).$$给定初值$a_0=5,a_1=7,a_2=8$,求$x_n$的通项.


解.我们先求矩阵的Jordan标准型,得到$$M = \left( {\begin{array}{*{20}{c}}3&{ - 2}&1\\4&{ - 1}&0\\4&{ - 3}&2\end{array}} \right) = SJ{S^{ - 1}} = \left( {\begin{array}{*{20}{c}}1&{1/2}&3\\2&0&4\\2&0&5\end{array}} \right)\left( {\begin{array}{*{20}{c}}1&1&0\\0&1&0\\0&0&2\end{array}} \right)\left( {\begin{array}{*{20}{c}}0&{5/2}&{ - 2}\\2&1&{ - 2}\\0&{ - 1}&1\end{array}} \right).$$因此

\begin{align*}\left( {\begin{array}{*{20}{c}}{{x_{3n - 3}}}\\{{x_{3n - 2}}}\\{{x_{3n - 1}}}\end{array}} \right) &= {\left( {\begin{array}{*{20}{c}}3&{ - 2}&1\\4&{ - 1}&0\\4&{ - 3}&2\end{array}} \right)^{n - 1}}\left( {\begin{array}{*{20}{c}}{{x_0}}\\{{x_1}}\\{{x_2}}\end{array}} \right) = \left( {\begin{array}{*{20}{c}}1&{1/2}&3\\2&0&4\\2&0&5\end{array}} \right){\left( {\begin{array}{*{20}{c}}1&1&0\\0&1&0\\0&0&2\end{array}} \right)^{n - 1}}\left( {\begin{array}{*{20}{c}}0&{5/2}&{ - 2}\\2&1&{ - 2}\\0&{ - 1}&1\end{array}} \right)\left( {\begin{array}{*{20}{c}}5\\7\\8\end{array}} \right)\\&= \left( {\begin{array}{*{20}{c}}1&{1/2}&3\\2&0&4\\2&0&5\end{array}} \right)\left( {\begin{array}{*{20}{c}}1&{n - 1}&0\\0&1&0\\0&0&{{2^{n - 1}}}\end{array}} \right)\left( {\begin{array}{*{20}{c}}0&{5/2}&{ - 2}\\2&1&{ - 2}\\0&{ - 1}&1\end{array}} \right)\left( {\begin{array}{*{20}{c}}5\\7\\8\end{array}} \right) = \left( {\begin{array}{*{20}{c}}{n + 1 + 3 \times {2^{n - 1}}}\\{2n + 1 + {2^{n + 1}}}\\{2n + 1 + 5 \times {2^{n - 1}}}\end{array}} \right).\end{align*}

空间中四点$O,A,B,C$使得\[\angle AOB=\frac{\pi}{2},\angle BOC=\frac{\pi}{3},\angle COA=\frac{\pi}{4}\]
设$AOB$决定的平面为$\pi_1$,$BOC$决定的平面为$\pi_2$,求$\pi_1,\pi_2$二面角.求出二面角的余弦值即可.

解.


设线性空间$V$, $\delta$是线性映射,其中向量$\beta$是$\delta$以特征值$\lambda$的特征向量.求证:对任意不全为零的$k_i(1\leq i \leq n)$,都存一组$V$的基使得$\beta$可以表示为该组基以$k_i$的线性组合.
 

证明存在矩阵$A$使得, $Aa_1=b_1,Aa_2=b_2$,其中$a_1\neq a_2,b_1\neq b_2$且$|A|=1$.

 

武汉大学2016年基础数学复试笔试试题

武汉大学2016年基础数学复试笔试试题

编辑整理:曹匹诺,Eufisky(Xiongge)

 

 

1.设$f(x)$在$(a,b)$上可微,且$f(x)$在$a$点右连续,试证:

(1) 若导函数$f'(x)$的右函数极限存在且为$A$,证明导函数$f'(x)$在$a$点的右侧存在且\[{{f'}_ + }\left( a \right) = \mathop {\lim }\limits_{x \to {a^ + }} f'\left( x \right) = A.\]

(2) $f'(x)$在$(a,b)$上不存在第一类间断点.

2.若级数$\sum_{n=1}^\infty a_n$收敛,证明级数\[\sum\limits_{n = 1}^\infty {\frac{{{a_n}}}{{\sqrt {{r_{n - 1}}} + \sqrt {{r_n}} }}} \]收敛,其中${r_n} = \sum_{k = n + 1}^\infty {{a_k}}$.

3.讨论$\lambda$取何值时, $y''+\lambda y=0$有非零的初值解,其中$y(0)=y(1)=0$.

4.$A$为正定矩阵, $A-B$为半正定矩阵,试证明:

(1) 方程$|\lambda B-A|=0$关于根$\lambda\geq1$;

(2) $|B|\leq |A|$.

5.讨论积分$\int_0^1 x^{p-1}\ln^2 xdx$在下列情况下的一致收敛性.

(1) $p\geq p_0>0$;

 

(2) $p>0$..

6. 设非负函数$f(x,y)$在区域$D$上可积,证明积分$\iint\limits_D f(x,y)dx=0$充分必要条件为$f(x,y)$在$D$上的连续点上等于$0$.

武汉大学2015年基础数学复试笔试试题

武汉大学2015年基础数学复试笔试试题

1.导函数极限定理, $f'(0)$存在, 而$\lim_{x\to0} f'(x)$不存在的例子.

事实上,可以考察

\[f\left( x \right) = \begin{cases}{x^2}\sin \frac{1}{x}, &x \ne 0\\0, &x = 0\end{cases}.\]

 

2.$\{a_n\}$是正项数列且单增.证明: $\sum_{n = 1}^\infty {\left( {\frac{{{a_{n + 1}}}}{{{a_n}}} - 1} \right)}$收敛$\Leftrightarrow$ $\{a_n\}$有界.

 

3.设$A$是$n$阶可逆复方阵,证明存在分解

\[A=UT,\]

其中$U$是酉矩阵,$T$是主对角线上都是正数的上三角型矩阵,并证明这种分解的唯一性。

 

4.讨论微分方程过点y=0的解的存在性和唯一性,其中$\alpha>0$.

\[\frac{dy}{dx}=|y|^{\alpha}.\]

 

5.证明含参变量积分

\[\int_{0}^{+\infty}\frac{\sin{xy}}{y(1+x)}dy\]

关于$x$在$0<\delta\leq x<+\infty$上一致收敛,在$0<x<+\infty$上非一致收敛。

 

 

6.利用数学归纳法证明$n$维空间中的$n+1$面体${B_{n + 1}}:\sum\limits_{i = 1}^n {{{\left( {{x_i}} \right)}^{1/\alpha }}} \le 1,\alpha > 0$的体积为\[V = \frac{{{2^n}{\alpha ^{n - 1}}{{\left[ {\Gamma \left( \alpha \right)} \right]}^{n - 1}}\Gamma \left( {\alpha + 1} \right)}}{{\Gamma \left( {\alpha n + 1} \right)}},\]

其中$\Gamma$为伽马函数.

 

2016数学系考研调剂信息

1.山东大学---徐老师:0531-88364197,网站:http://www.yz.sdu.edu.cn/getNewsDetail.site?newsId=200330f8-6e1c-45c2-b4bb-65262be64252

参考:http://www.zhihu.com/question/28896781

2.

某家公司的笔试题

1.设实数列$\{a_n\}$满足$a_{n+p}\leq a_{n}+a_{p}$对于任意的正整数$p,n$,证明:$$\lim\limits_{n\to +\infty}\dfrac{a_n}{n}=\inf\limits_n \dfrac{a_n}{n}.$$ 
2.设实函数$f(x)$在$(0,1)$内一阶可导且满足$f(1)=1,f(0)=0$,设
\begin{equation*}\int_0^1|f'(x)-f(x)|\geq {u}.\end{equation*}求$u$的最大值。
3.给定一个圆求在这个圆里面随机选择四个点围成一个凸集的概率。
 
来源:http://www.mysanco.cn/wenda/index.php?class=discuss&action=question_item&questionid=7170

2013武大数分压轴题

(13年武大数分)求$\displaystyle I = \iint\limits_\Sigma  {{{\left( {{x^2} + {y^2} + {z^2}} \right)}^{ - \frac{3}{2}}}{{\left( {\frac{{{x^2}}}{{{a^4}}} + \frac{{{y^2}}}{{{b^4}}} + \frac{{{z^2}}}{{{c^4}}}} \right)}^{ - \frac{1}{2}}}dS} $,其中$\sum$为椭球面: $\displaystyle \frac{{{x^2}}}{{{a^2}}} + \frac{{{y^2}}}{{{b^2}}} + \frac{{{z^2}}}{{{c^2}}} = 1(a,b,c>0)$.


下面是自己的解答:

令$x = a\sin \varphi \cos \theta ,y = b\sin \varphi \sin \theta ,z = c\cos \varphi $,其中$0\leq \theta\leq 2\pi,0\leq\varphi \leq\pi$,经计算得到
\[\frac{{\partial \left( {y,z} \right)}}{{\partial \left( {\varphi ,\theta } \right)}} = bc{\sin ^2}\varphi \cos \theta ,\frac{{\partial \left( {z,x} \right)}}{{\partial \left( {\varphi ,\theta } \right)}} = ac{\sin ^2}\varphi \sin \theta ,\frac{{\partial \left( {x,y} \right)}}{{\partial \left( {\varphi ,\theta } \right)}} = ab\sin \varphi \cos \varphi ,\]
所以
\begin{align*}EG - {F^2} &= {\left( {\frac{{\partial \left( {y,z} \right)}}{{\partial \left( {\varphi ,\theta } \right)}}} \right)^2} + {\left( {\frac{{\partial \left( {z,x} \right)}}{{\partial \left( {\varphi ,\theta } \right)}}} \right)^2} + {\left( {\frac{{\partial \left( {x,y} \right)}}{{\partial \left( {\varphi ,\theta } \right)}}} \right)^2}\\& = {\left( {abc} \right)^2}{\sin ^2}\varphi \left( {\frac{{{{\sin }^2}\varphi {{\cos }^2}\theta }}{{{a^2}}} + \frac{{{{\sin }^2}\varphi {{\sin }^2}\theta }}{{{b^2}}} + \frac{{{{\cos }^2}\varphi }}{{{c^2}}}} \right).\end{align*}
而这时被积函数化为
\begin{align*}&{\left( {{x^2} + {y^2} + {z^2}} \right)^{ - \frac{3}{2}}}{\left( {\frac{{{x^2}}}{{{a^4}}} + \frac{{{y^2}}}{{{b^4}}} + \frac{{{z^2}}}{{{c^4}}}} \right)^{ - \frac{1}{2}}}\\= &{\left( {{a^2}{{\sin }^2}\varphi {{\cos }^2}\theta  + {b^2}{{\sin }^2}\varphi {{\sin }^2}\theta  + {c^2}{{\cos }^2}\varphi } \right)^{ - \frac{3}{2}}}\\&{\left( {\frac{{{{\sin }^2}\varphi {{\cos }^2}\theta }}{{{a^2}}} + \frac{{{{\sin }^2}\varphi {{\sin }^2}\theta }}{{{b^2}}} + \frac{{{{\cos }^2}\varphi }}{{{c^2}}}} \right)^{ - \frac{1}{2}}}.\end{align*}
因此
\[I = abc\iint\limits_{\left[ {0,\pi } \right] \times \left[ {0,2\pi } \right]} {{{\left( {{a^2}{{\sin }^2}\varphi {{\cos }^2}\theta  + {b^2}{{\sin }^2}\varphi {{\sin }^2}\theta  + {c^2}{{\cos }^2}\varphi } \right)}^{ - \frac{3}{2}}}\sin \varphi d\varphi d\theta } \]
 
注意到这么一个事实,当$M+Nx^2$不取$0$且$M\neq 0$时,我们有
\[\int {{{\left( {M + N{x^2}} \right)}^{ - 3/2}}dx}  = \frac{1}{M} \cdot \frac{x}{{\sqrt {M + N{x^2}} }} + C.\]
 
\begin{align*}I &= abc\int_0^{2\pi } {d\theta } \int_0^\pi  {{{\left( {{a^2}{{\sin }^2}\varphi {{\cos }^2}\theta  + {b^2}{{\sin }^2}\varphi {{\sin }^2}\theta  + {c^2}{{\cos }^2}\varphi } \right)}^{ - \frac{3}{2}}}\sin \varphi d\varphi } \\&=  - abc\int_0^{2\pi } {d\theta } \int_0^\pi  {{{\left( {{a^2}{{\sin }^2}\varphi {{\cos }^2}\theta  + {b^2}{{\sin }^2}\varphi {{\sin }^2}\theta  + {c^2}{{\cos }^2}\varphi } \right)}^{ - \frac{3}{2}}}d\left( {\cos \varphi } \right)} \\&=  - abc\int_0^{2\pi } {d\theta } \int_0^\pi  {{{\left[ {\left( {{a^2}{{\cos }^2}\theta  + {b^2}{{\sin }^2}\theta } \right) + \left( {{c^2} - {a^2}{{\cos }^2}\theta  - {b^2}{{\sin }^2}\theta } \right){{\cos }^2}\varphi } \right]}^{ - \frac{3}{2}}}d\left( {\cos \varphi } \right)} \\&= abc\int_0^{2\pi } {d\theta } \int_{ - 1}^1 {{{\left[ {\left( {{a^2}{{\cos }^2}\theta  + {b^2}{{\sin }^2}\theta } \right) + \left( {{c^2} - {a^2}{{\cos }^2}\theta  - {b^2}{{\sin }^2}\theta } \right){x^2}} \right]}^{ - \frac{3}{2}}}dx} \\&= abc\int_0^{2\pi } {\frac{2}{{\left( {{a^2}{{\cos }^2}\theta  + {b^2}{{\sin }^2}\theta } \right)c}}d\theta }  = 4ab\int_0^\pi  {\frac{1}{{{a^2}{{\cos }^2}\theta  + {b^2}{{\sin }^2}\theta }}d\theta } .\end{align*}
\begin{align*}&\int_0^\pi  {\frac{1}{{{a^2}{{\cos }^2}\theta  + {b^2}{{\sin }^2}\theta }}d\theta }  = \int_0^{\frac{\pi }{2}} {\frac{1}{{{a^2}{{\cos }^2}\theta  + {b^2}{{\sin }^2}\theta }}d\theta }  + \int_{\frac{\pi }{2}}^\pi  {\frac{1}{{{a^2}{{\cos }^2}\theta  + {b^2}{{\sin }^2}\theta }}d\theta } \\= &\int_0^{\frac{\pi }{2}} {\frac{1}{{{a^2}{{\cos }^2}\theta  + {b^2}{{\sin }^2}\theta }}d\theta }  + \int_0^{\frac{\pi }{2}} {\frac{1}{{{a^2}{{\sin }^2}\theta  + {b^2}{{\cos }^2}\theta }}d\theta } \\= &\int_0^{ + \infty } {\frac{1}{{{a^2} + {b^2}{x^2}}}d}  + \int_0^{ + \infty } {\frac{1}{{{a^2}{x^2} + {b^2}}}dx}  = \frac{1}{{ab}}\left. {\arctan \left( {\frac{b}{a}x} \right)} \right|_0^{ + \infty } + \frac{1}{{ab}}\left. {\arctan \left( {\frac{a}{b}x} \right)} \right|_0^{ + \infty }\\= &\frac{\pi }{{ab}}.\end{align*}
进而得到
\[I = 4ab\int_0^\pi  {\frac{1}{{{a^2}{{\cos }^2}\theta  + {b^2}{{\sin }^2}\theta }}d\theta }  = 4\pi .\]

另外有更好的方法:(Hansschwarzkopf)

注意到$\Sigma$ 在点$(x,y,z)$处的单位外法向量是
$$n=\frac{\left(\frac{x}{a^2},\frac{y}{b^2},\frac{z}{c^2}\right)}{\sqrt{\frac{x^2}{a^4}+\frac{y^2}{b^4}+\frac{z^2}{c^4}}},$$
且$1=x\cdot \frac{x}{a^2}+y\cdot\frac{y}{b^2}+z\cdot\frac{z}{c^2}$.
从而原积分可写成第二型曲面积分
$$\iint\limits_\Sigma \frac{xdydz+yd zd x+zdxdy}{\sqrt{(x^2+y^2+z^2)^3}}.$$
作小球面$S_\varepsilon: x^2+y^2+z^2=\varepsilon^2$. 运用Gauss公式可知
$$\iint\limits_\Sigma \frac{xd yd z+yd zd x+zd xd y}{\sqrt{(x^2+y^2+z^2)^3}} =\iint\limits_{S_\varepsilon} \frac{xdyd z+ydzd x+zd xd y}{\sqrt{(x^2+y^2+z^2)^3}}=4\pi.$$ 即
$$\iint\limits_\Sigma\frac{d S}{\sqrt{(x^2+y^2+z^2)^3}\sqrt{\frac{x^2}{a^4}+\frac{y^2}{b^4}+\frac{z^2}{c^4}}}=4\pi.$$

 

2011年南开大学高等代数试题

 

参考:http://www.math.org.cn/forum.php?mod=viewthread&tid=21850&extra=&page=2

2014年浙江大学数学分析考研试题解答

6.设空间体积为$V$的任意$\Omega,X_0\in \Omega ,0<\alpha<3$.证明

\[\int_\Omega {{{\left| {X - {X_0}} \right|}^{\alpha - 3}}dX} \le C{V^{\alpha /3}}, \text{其中$C$只与$\alpha$有关}.\]


enlightened证:(Veer)由于$\alpha-3>-3$且$\displaystyle \int_\Omega {{{\left| {X - {X_0}} \right|}^{\alpha - 3}}dX}$为三重积分,故积分广义可积.

 

在$X_0$处作一以$X_0$为圆心的球$D$,使其体积为$V_D=V$.设$D$的半径为$R$.记$D_1=D\cap \Omega,D_2=D/D_1,\Omega_2=\Omega/D_1$,则易知$V_{D_2}=V_{\Omega_2}$.显然

\begin{align*}\int_D {{{\left| {X - {X_0}} \right|}^{\alpha - 3}}dX} &= \int_{{D_1}} {{{\left| {X - {X_0}} \right|}^{\alpha - 3}}dX} + \int_{{D_2}} {{{\left| {X - {X_0}} \right|}^{\alpha - 3}}dX} \\\int_\Omega {{{\left| {X - {X_0}} \right|}^{\alpha - 3}}dX} &= \int_{{D_1}} {{{\left| {X - {X_0}} \right|}^{\alpha - 3}}dX} + \int_{{\Omega _2}} {{{\left| {X - {X_0}} \right|}^{\alpha - 3}}dX} .\end{align*}

由积分中值定理有

\begin{align*}\int_{{D_2}} {{{\left| {X - {X_0}} \right|}^{\alpha - 3}}dX} &= {\left| {\xi - {X_0}} \right|^{\alpha - 3}}{V_{{D_2}}},\xi \in {D_2}\\\int_{{\Omega _2}} {{{\left| {X - {X_0}} \right|}^{\alpha - 3}}dX} &= {\left| {\eta - {X_0}} \right|^{\alpha - 3}}{V_{{\Omega_2}}},\eta \in {\Omega _2}.\end{align*}

易知${\left| {\xi - {X_0}} \right|^{\alpha - 3}} \ge {\left| {\eta - {X_0}} \right|^{\alpha - 3}}$.又因为${V_{{D_2}}} = {V_{{\Omega _2}}}$,则\[\int_\Omega {{{\left| {X - {X_0}} \right|}^{\alpha - 3}}dX} \le \int_D {{{\left| {X - {X_0}} \right|}^{\alpha - 3}}dX} .\]由球坐标变换易得\[\int_D {{{\left| {X - {X_0}} \right|}^{\alpha - 3}}dX} = \int_0^\pi {d\varphi } \int_0^{2\pi } {d\theta } \int_0^R {{r^{\alpha - 1}}\sin \varphi dr} = 4\pi \frac{{{R^\alpha }}}{\alpha }.\]又因为$\displaystyle {V_D} = V = \frac{4}{3}\pi {R^3}$,则\[\int_D {{{\left| {X - {X_0}} \right|}^{\alpha - 3}}dX} = \frac{{{3^{\alpha /3}}{{\left( {4\pi } \right)}^{1 - \alpha /3}}}}{\alpha }{V^{\alpha /3}}.\]故\[\int_\Omega {{{\left| {X - {X_0}} \right|}^{\alpha - 3}}dX} \le \frac{{{3^{\alpha /3}}{{\left( {4\pi } \right)}^{1 - \alpha /3}}}}{\alpha }{V^{\alpha /3}},\]取$\displaystyle C = \frac{{{3^{\alpha /3}}{{\left( {4\pi } \right)}^{1 - \alpha /3}}}}{\alpha }$.

 

注:从上可以看出当$\Omega=D$时不等式可取等号,故$C$是最佳的,且此题可推广到$n$维上.

 


7.$f(x)$在$[0,1]$单增,证明:

\[\mathop {\lim }\limits_{y \to + \infty } \int_0^1 {f\left( x \right)\frac{{\sin xy}}{x}dx} = \frac{\pi }{2}f\left( {{0_ + }} \right).\]


enlightened证:这是Dirichlet引理,菲赫金哥尔茨的《微积分教程》第三卷P358有详细的证明.另外,汪林的《数学分析问题研究与评注》P147上有他的推广及其证明.

 

对任意给出的$\varepsilon>0$, $\exists 0<\delta<1$,使得对于$0<t\leq \delta$,

\[0 \le g\left( t \right) - g\left( {{0_ + }} \right) < M_1\varepsilon ,\]

其中$M_1$是任意给定的常数.

 

考察积分

\begin{align*}\int_0^1 {\left[ {f\left( x \right) - f\left( {{0_ + }} \right)} \right]\frac{{\sin xy}}{x}dx} &= \left( {\int_0^\delta {} + \int_\delta ^1 {} } \right)\left[ {f\left( x \right) - f\left( {{0_ + }} \right)} \right]\frac{{\sin xy}}{x}dx\\&= {I_1} + {I_2}.\end{align*}

 

对于$I_1$,运用积分第二中值定理,我们有

\[{I_1} = \left[ {f\left( \delta \right) - f\left( {{0_ + }} \right)} \right]\int_\eta ^\delta {\frac{{\sin xy}}{x}dx} = \left[ {f\left( \delta \right) - f\left( {{0_ + }} \right)} \right]\int_{y\eta }^{y\delta } {\frac{{\sin z}}{z}dz} ,\]

 

其中第二个因子对于一切值$y$一致有界.事实上,由反常积分$\displaystyle \int_0^\infty {\frac{{\sin z}}{z}dz}$的收敛性,可见当$z\to \infty$时, $z(z\geq 0)$的连续函数$\displaystyle \int_0^z {\frac{{\sin z}}{z}dz} $有有限的极限,并且对于一切值$z$有界

\[\left| {\int_0^z {\frac{{\sin z}}{z}dz} } \right| \le L\left( L \text{为常数}\right),\]从而

\[\left| {\int_{y\eta }^{y\delta } {\frac{{\sin z}}{z}dz} } \right| = \left| {\int_0^{y\delta } {} + \int_0^{y\eta } {} } \right| \le 2L.\]

 

对于第一个因子,取$M_1=\frac{1 }{{4L}}$,则有$f\left( \delta \right) - f\left( {{0_ + }} \right) < \frac{\varepsilon }{{4L}}$.

 

 

因此\[\left| {{I_1}} \right| \le \left[ {f\left( \delta \right) - f\left( {{0_ + }} \right)} \right]\left| {\int_{y\eta }^{y\delta } {\frac{{\sin z}}{z}dz} } \right| < \frac{\varepsilon }{{4L}} \cdot 2L = \frac{\varepsilon }{2}.\]

 

至于$I_2$,由于$\displaystyle \int_\delta ^1 {\frac{{f\left( x \right) - f\left( {{0_ + }} \right)}}{x}dx} $存在,由Riemann-Lebesgue引理可知$\mathop {\lim }\limits_{y \to \infty } {I_2} = 0$,即对$\varepsilon >0,\exists M_2>0$,使得$y>M_2$时,有$\left| {{I_2}} \right| < \frac{\varepsilon }{2}$.

 

因此\[\left| {\int_0^1 {\left[ {f\left( x \right) - f\left( {{0_ + }} \right)} \right]\frac{{\sin xy}}{x}dx} } \right| \le \left| {{I_1}} \right| + \left| {{I_2}} \right| < \varepsilon .\]

即\[\mathop {\lim }\limits_{y \to + \infty } \int_0^1 {\left[ {f\left( x \right) - f\left( {{0_ + }} \right)} \right]\frac{{\sin xy}}{x}dx} = 0.\]

 

从而

\begin{align*}&\mathop {\lim }\limits_{y \to + \infty } \int_0^1 {f\left( x \right)\frac{{\sin xy}}{x}dx} = \frac{\pi }{2}f\left( {{0_ + }} \right)\\=& \mathop {\lim }\limits_{y \to + \infty } \int_0^1 {\left[ {f\left( x \right) - f\left( {{0_ + }} \right)} \right]\frac{{\sin xy}}{x}dx} + f\left( {{0_ + }} \right)\mathop {\lim }\limits_{y \to + \infty } \int_0^1 {\frac{{\sin xy}}{x}dx} \\= &0 + f\left( {{0_ + }} \right)\int_0^{ + \infty } {\frac{{\sin z}}{z}dz} = \frac{\pi }{2}f\left( {{0_ + }} \right).\end{align*}